please help me!

find x​

Please Help Me! Find X

Answers

Answer 1

Answer:

√13 miles

Step-by-step explanation:

Using Pythagorean Theorem,

h^2=p^2+b^2

Here,

h=x

p=2miles( taking the angle opposite to 2miles as reference)

b=3miles

Now,

x^2=2^2+3^2

x=√(4+9)

x=√13 miles

Answer 2
For this question x=13miles

Related Questions

The following argument is an instance of one of the five inference forms MP, MT, HS, DS, Conj. Identify the form.

[(P ≡ T) • (H • N)] ⊃ (T ⊃ ~S)
(T ⊃ ~S) ⊃ [(H ∨ E) ∨ R]
[(P ≡ T) • (H • N)] ⊃ [(H ∨ E) ∨ R]

a. MP
b. DS
c. MT
d. Conj
e. HS

Answers

Answer:

e. HS

Step-by-step explanation:

The argument:

[(P ≡ T) • (H • N)] ⊃ (T ⊃ ~S)

(T ⊃ ~S) ⊃ [(H ∨ E) ∨ R]

[(P ≡ T) • (H • N)] ⊃ [(H ∨ E) ∨ R]

is an instance of one of hypothetical syllogism (HS).

Hypothetical syllogism contains conditional statements for its premises.

Let

p = [(P ≡ T) • (H • N)]

q = (T ⊃ ~S)

r = [(H ∨ E) ∨ R]

The this can be interpreted as:

p ⊃ q

q ⊃ r

p ⊃ r

This interprets that:

If p then q

but if q then r

therefore if p then r

Thus, in logic HS is a valid argument form:

p → q

q → r

∴ p → r

Note that ⊃ symbol is used to symbolize implication relationships. This is used in conditional statements which are represented in the if...then... form.  For example p ⊃ q means: if p then q. So the type of Hypothetical syllogism used in this is conditional syllogism.

There are three parts of syllogism:

major premise

minor premise

conclusion

An example is:

If ABC is hardworking, then ABC will go to a good college.  

Major premise: ABC is hardworking.

Minor premise: Because ABC is hardworking , ABC will score well.

Conclusion: ABC will go to a good college.

Example of Hypothetical syllogism:

If AB is a CD, then EF is a GH

if WX is a YZ, then AB is a CD

therefore if WX is a YZ, then EF is a GH

This can be understood with the help of an example:

If you study the topic, then you will understand the topic.  

If you understand the topic, then you will pass the quiz.

Therefore, if you study the topic, then you will pass the quiz.

The probability distribution of a random variable X is given. x 1 2 3 4 P(X = x) 0.4 0.1 0.3 0.2 Compute the mean, variance, and standard deviation of X. (Round your answers to two decimal places.) mean variance standard deviation

Answers

Mean:

[tex]E(X) = \displaystyle \sum_{x\in\{1,2,3,4\}}x\,P(X=x) = 1\times0.4 + 2\times0.1 + 3\times0.3 + 4\times0.2 = \boxed{2.3}[/tex]

Variance:

[tex]\displaystyle V(X) = E\left((X-E(X))^2\right) = E(X^2) - E(X)^2 \\\\ E(X^2) = \sum_{x\in\{1,2,3,4\}}x^2\,P(X=x) = 1^2\times0.4 + 2^2\times0.1 + 3^2\times0.3 + 4^2\times0.2 = 6.7 \\\\ \implies V(X) = 6.7 - 2.3^2 = \boxed{1.41}[/tex]

Standard deviation:

[tex]\sigma_X = \sqrt{V(X)} = \sqrt{1.41} \approx \boxed{1.19}[/tex]

The domain and range for function g are D(−infinity symbol, infinity symbol) and R(−infinity symbol, infinity symbol). Describe the following statement:

the limit as x approaches 3 of the function g of x equals 4

Select one:
a. The value of g at 3 is 4.
b. The value of g at 4 is 3.
c. As x gets closer to 4, the value of g gets closer to 3.
d. As x gets closer to 3, the value of g gets closer to 4.

Answers

The answer is d: As x gets closer to 3, the value of g gets closer to 4.

The limit of a function h of x as  x approaches the value a, written as [tex]\lim_{x \to a} h(x) = L[/tex] is the value the function h(x) approaches as x tends to the value "a", written as x → a. In this case, L.

Given the domain and range for function g are D(−∞, ∞) and R(−∞, ∞) and that the limit as x approaches 3 of the function g of x equals 4.

This implies that as x gets closer and closer to 3, the value of g gets closer and closer to 4.

Since the value g gets closer to is 3 as x gets closer to 4, we can write that the limit as x approaches 3 of the function g of x equals 4.

we can write this as

[tex]\lim_{x \to 3} g(x) = 4[/tex]

So, as x gets closer to 3, the value of g gets closer to 4.

So, the answer is d: As x gets closer to 3, the value of g gets closer to 4.

Learn more about limits here:

https://brainly.com/question/23144996

sorry to keep asking questions

Answers

Answer:

y = [tex]\sqrt[3]{x-5}[/tex]

Step-by-step explanation:

To find the inverse of any function you basically switch x and y.

function = y = x^3 + 5

Now we switch x and y

x = y^3 +5

Solve for y,

x - 5 = y^3

switch sides,

y^3 = x-5

y = [tex]\sqrt[3]{x-5}[/tex]

Answer:

[tex]\Large \boxed{{f^{-1}(x)=\sqrt[3]{x-5}}}[/tex]

Step-by-step explanation:

The function is given,

[tex]f(x)=x^3 +5[/tex]

The inverse of a function reverses the original function.

Replace f(x) with y.

[tex]y=x^3 +5[/tex]

Switch variables.

[tex]x=y^3 +5[/tex]

Solve for y to find the inverse.

Subtract 5 from both sides.

[tex]x-5=y^3[/tex]

Take the cube root of both sides.

[tex]\sqrt[3]{x-5} =y[/tex]

What is the following simplified product? Assume x greater-than-or-equal-to 0 (StartRoot 10 x Superscript 4 Baseline EndRoot minus x StarRoot 5 x squared EndRoot) (2 StartRoot 15 x Superscript 4 Baseline EndRoot + StartRoot 3 x cubed EndRoot) 10 x Superscript 4 Baseline StartRoot 6 EndRoot + x cubed StartRoot 30 x EndRoot minus 10 x Superscript 4 Baseline StartRoot 3 EndRoot + x squared StartRoot 15 x EndRoot 11 x Superscript 4 Baseline StartRoot 6 EndRoot + x cubed StartRoot 30 x EndRoot minus x Superscript 4 Baseline StartRoot 75 EndRoot + x squared StartRoot 15 EndRoot 10 x Superscript 4 Baseline StartRoot 6 EndRoot + x cubed StartRoot 30 x EndRoot minus 10 x Superscript 4 Baseline StartRoot 3 EndRoot minus x squared StartRoot 15 EndRoot 11 x Superscript 4 Baseline StartRoot 6 EndRoot + x cubed StartRoot 30 x EndRoot minus 10 x Superscript 4 Baseline StartRoot 3 EndRoot minus x cubed StartRoot 15 x EndRoot

Answers

Answer:

[tex]\bold{10x^4\sqrt{6}+x^3\sqrt{30x}-10x^4\sqrt{3}-x^3\sqrt{15x}}[/tex]

Step-by-step explanation:

To find:

Simplified product of:

[tex](\sqrt{10x^4}-x\sqrt{5x^2})(2\sqrt{15x^4}+\sqrt{3x^3})[/tex]

Solution:

First of all, let us have a look at some of the formula:

1. [tex](a+b) (c+d) = ac+bc+ad+bd[/tex]

2. [tex]a^b\times a^c =a^{b+c }[/tex]

3. [tex]\sqrt{a^{2b}} = \sqrt{a^b.a^b}=a^b[/tex]

4. [tex]\sqrt a \times \sqrt b = \sqrt{a\times b}[/tex]

Now, let us apply the above formula to solve the given expression.

[tex](\sqrt{10x^4}-x\sqrt{5x^2})(2\sqrt{15x^4}+\sqrt{3x^3})\\\\\Rightarrow(\sqrt{10x^4})(2\sqrt{15x^4})+(\sqrt{10x^4})(\sqrt{3x^3})-(x\sqrt{5x^2})(2\sqrt{15x^4})-(x\sqrt{5x^2})(\sqrt{3x^3})\\\\\Rightarrow2\sqrt{150x^8}+\sqrt{30x^7}-2x\sqrt{75x^6}-x\sqrt{15x^5}\\\\\Rightarrow\bold{10x^4\sqrt{6}+x^3\sqrt{30x}-10x^4\sqrt{3}-x^3\sqrt{15x}}[/tex]

The answer is:

[tex]\bold{10x^4\sqrt{6}+x^3\sqrt{30x}-10x^4\sqrt{3}-x^3\sqrt{15x}}[/tex]

Answer:

Its D

Step-by-step explanation:

Tia uses 3/4 cup of pumpkin to make 1 1/4 pounds of dog treats. How much pumpkin does Tia use to make 1 pound of treats?

Answers

Answer:

4/5 cups  to make 1 pound of dog treats

Step-by-step explanation:

3/4 cups : 1 1/4 pounds

x cups : 1 pound

Cross multiply

3/4 * 1  = 1 1/4  * x

x = 3/4 / 1 1/4

= 3/4  /  5/4

= 3/4 * 4/5

= 3/5 cups

The black graph is the graph of
y = f(x). Choose the equation for the
red graph.
a. y = f(2x)
b. y=f)
C. 2y = f(x)
d. = f(x)

Answers

9514 1404 393

Answer:

  c. 2y = f(x)

Step-by-step explanation:

Each point on the red graph is half the distance from the x-axis as the same point on the black graph. That is, the vertical scale factor is 1/2:

  y = (1/2)f(x)

Multiplying this equation by 2 gives one that matches an answer choice:

  2y = f(x)

rmined that her father’s age is four less than three times her age. If a represents Marcy’s age, which expression represents her fathers age?

Answers

Answer:

3x - 4 is the expression that will represent her father's age

When x€Q, what is the solution of 3x-2/2=x-1/2 ?​

Answers

Answer:

x = [tex]\frac{1}{2}[/tex]

Step-by-step explanation:

[tex]\frac{3x-2}{2}[/tex] = [tex]\frac{x-1}{2}[/tex]

Cross-multiply:

2(3x-2) = 2(x-1)

Simplify:

6x - 4 = 2x - 2

Subtract 2x from both sides:

4x - 4 = -2

Add 4 to both sides:

4x = 2

Divide both sides by 4:

x = [tex]\frac{1}{2}[/tex]

Suppose h(x)=3x-2 and j(x) = ax +b. Find a relationship between a and b such that h(j(x)) = j(h(x))

Probably a simple answer, but I'm completely lost at what I'm being asked here.

Answers

Answer:

[tex]\displaystyle a = \frac{1}{3} \text{ and } b = \frac{2}{3}[/tex]

Step-by-step explanation:

We can use the definition of inverse functions. Recall that if two functions, f and g are inverses, then:

[tex]\displaystyle f(g(x)) = g(f(x)) = x[/tex]

So, we can let j be the inverse function of h.

Function h is given by:

[tex]\displaystyle h(x) = y = 3x-2[/tex]

Find its inverse. Flip variables:

[tex]x = 3y - 2[/tex]

Solve for y. Add:

[tex]\displaystyle x + 2 = 3y[/tex]

Hence:

[tex]\displaystyle h^{-1}(x) = j(x) = \frac{x+2}{3} = \frac{1}{3} x + \frac{2}{3}[/tex]

Therefore, a = 1/3 and b = 2/3.

We can verify our solution:

[tex]\displaystyle \begin{aligned} h(j(x)) &= h\left( \frac{1}{3} x + \frac{2}{3}\right) \\ \\ &= 3\left(\frac{1}{3}x + \frac{2}{3}\right) -2 \\ \\ &= (x + 2) -2 \\ \\ &= x \end{aligned}[/tex]

And:

[tex]\displaystyle \begin{aligned} j(h(x)) &= j\left(3x-2\right) \\ \\ &= \frac{1}{3}\left( 3x-2\right)+\frac{2}{3} \\ \\ &=\left( x- \frac{2}{3}\right) + \frac{2}{3} \\ \\ &= x \stackrel{\checkmark}{=} x\end{aligned}[/tex]

determine x in the following equation 2x - 4 = 10

Answers

Answer:

7

Step-by-step explanation:

10+4 = 14

14/2  = 7

x = 7

A random sample of 1400 Internet users was selected from the records of a large Internet provider and asked whether they would use the Internet or the library to obtain information about health issues. Of these, 872 said they would use the Internet

1. The standard error ˆp SE of the proportion pˆ that would use the Internet rather than the library is:_______

a. 0.013.
b. 0.25.
c. 0.485.
d. 0.623.

2. If the Internet provider wanted an estimate of the proportion p that would use the Internet rather than the library, with a margin of error of at most 0.02 in a 99% confidence interval, how large a sample size would be required? (Assume that we don’t have any prior information about p).

a. 33
b. 3909
c. 2401
d. 4161

Answers

Answer:

1   [tex]\sigma_{\= x } = 0.0130[/tex]

2  [tex]n = 3908.5[/tex]

Step-by-step explanation:

From the question we are told that

     The  sample size is  [tex]n_p = 1400[/tex]

      The  number of those that said the would use internet is [tex]k = 872[/tex]

       The margin of error is  [tex]E = 0.02[/tex]

Generally the sample proportion is mathematically evaluated as

            [tex]\r p = \frac{k}{n_p}[/tex]

substituting values

            [tex]\r p = \frac{ 872}{1400}[/tex]

substituting values

           [tex]\r p = 0.623[/tex]

Generally the standard error of  [tex]\r p[/tex] is mathematically evaluated as

           [tex]\sigma_{\= x } = \sqrt{\frac{\r p (1- \r p)}{n} }[/tex]

substituting values

              [tex]\sigma_{\= x } = \sqrt{\frac{0.623 (1- 0.623)}{1400} }[/tex]

              [tex]\sigma_{\= x } = 0.0130[/tex]

For  a  95% confidence interval the confidence level is  95%

Given that the confidence interval is 95% the we can evaluated the level of confidence as

                    [tex]\alpha = 100 - 99[/tex]

                   [tex]\alpha = 1\%[/tex]

                   [tex]\alpha = 0.01[/tex]

Next we obtain the critical value of  [tex]\frac{\alpha }{2}[/tex] from normal distribution table (reference math dot  armstrong dot edu) , the value is  

         [tex]Z_{\frac{\alpha }{2} } = 2.58[/tex]

Give that the population size is very large  the sample size is mathematically represented as

            [tex]n = [ \frac{Z_{\frac{\alpha }{2} ^2 * \r p ( 1 - \r p )}}{E^2} ][/tex]

substituting values  

             [tex]n = [ \frac{2.58 ^2 * 0.623 ( 1 -0.623 )}{0.02^2} ][/tex]

             [tex]n = 3908.5[/tex]

Which expression corresponds to this graph?

Answers

Answer: Choice A

The number line graph is visually showing every number that is 19 or smaller; hence [tex]x \le 19[/tex]

Note the use of a closed or filled in circle at the endpoint (in contrast to an open circle). This indicates we are including the endpoint 19 as part of the solution set, and that's why we go for "or equal to" as part of the inequality sign.

A circular pizza has a diameter of 12 inches and is cut into 10 congruent slices. What is the area of each slice, to the nearest tenth?
Group of answer choices

14.4 straight pi inches squared

3.6 straight pi inches squared

2.4 straight pi inches squared

12.4 straight pi inches squared

Answers

Diameter=12inradius=r=12/2=6in

[tex]\\ \sf\longmapsto Area=2\pi r[/tex]

[tex]\\ \sf\longmapsto Area=2(3.14)\times 6[/tex]

[tex]\\ \sf\longmapsto Area=2(18.84)[/tex]

[tex]\\ \sf\longmapsto Area=37.68in^2[/tex]

Now area of each slice

[tex]\\ \sf\longmapsto \dfrac{37.68}{12}[/tex]

[tex]\\ \sf\longmapsto 3.6in^2[/tex]

The area of each slice of the pizza will be 3.6π square inches. The correct option is B.

What is the area of the sector?

The circle is defined as the locus of the point traces around a fixed point called the center and is equidistant from the out trace.

It is given that a circular pizza has a diameter of 12 inches and is cut into 10 congruent slices.

Let r is the radius of the sector and θ be the angle subtends by the sector at the center. Then the area of the sector of the circle will be

Area = (θ/2π) πr²

angle = 2π/10

The area will be calculated as below:-

Area =  (2π/10)/2π) πr²

Area =  ( π x 6 x 6 ) / 10

Area = 3.6π square inches

Therefore, the area of each slice of the pizza will be 3.6π square inches. The correct option is B.

More about the area of the sector link is given below.

https://brainly.com/question/7512468

#SPJ2

the length of a triangle is x and its width is 2x. what is the area if the length and width are each increased by 1?
A. 2x^2+ 3x+ 1
B. 2x^2+ 1
C. 2x^2+ 2x+ 1
D. 2x^2+ 3x+ 2

Answers

The answer to the question is B.

Answer:

Hey there!

(2x+1)(x+1)

2x^2+1x+2x+1

2x^2+3x+1

The answer would be A.

Let me know if this helps :)

Addition prop of equality
subtraction prop of quality
multiplication prop of equality
Division prop of equality
simplifying
distrib prop

Answers

1 multiplication prop
2simplifying
3 Addition prop
4 simplifying

Which equation has the graph shown below? (0, 3) (2,0) ​

Answers

Answer:

6x+4y=12

Step-by-step explanation:

when x-intercept is 0

6(0)+4y=12

0+4y=12

divide by 4 both side

y=3

when y-intercept is 0

6x+4(0)=12

6x+0=12

6x=12

divide by 6 both side

x=2

Choose the correct ray whose endpoint is B.

Answers

Answer:

The second option.

Step-by-step explanation:

The first option consists of a line that extends at both opposite sides to infinity, with no precise end.

The third option is a ray that has an endpoint of A, and extends to infinity towards B.

The fourth option is a line segment. It has two endpoints, B and A.

The second portion is a ray that has an endpoint B, and extends towards A in one direction, to infinity.

The answer is the 2nd option.

Please answer! I am struggling with this question! Please show ALL work! <3 (the answer choices are provided on a separate image)

Answers

Answer:

The radius is 18 inches

Step-by-step explanation:

The circumference of a circle is given by

C = 2 * pi *r

36 pi = 2 * pi *r

Divide each side by pi

36 = 2r

Divide each side by 2

18 =r

Answer:

The answer is option C

Step-by-step explanation:

Circumference of a circle = 2πr

where

r is the radius of the circle

From the question

Circumference = 36π inches

To find the radius substitute the value of the circumference into the above formula and solve for the radius

That's

[tex]36\pi = 2\pi r[/tex]

Divide both sides by 2π

We have

[tex] \frac{36\pi}{2\pi} = \frac{2\pi \: r}{2\pi} [/tex]

We have the final answer as

r = 18 inches

Hope this helps you

Evaluate 26 + a if a = 8

Answers

Answer:

34

Step-by-step explanation:

= 26 + 8

= 34

About 10% of the human population is left-handed. Suppose a researcher speculates that artists are more likely to be left-handed than other people in the general population. The researcher surveys 200 artists and finds that 26 of them are left-handed.Required:a. Define the parameter of interest and give the null value.b. State the researcher's null and alternative hypotheses.c. What proportion of the sample of artists is left-handed?d. To calculate a p-value for the hypothesis test, what probability should the researcher calculate? Make your answer specific to this situation.

Answers

Answer:

Given the information in the question;

a) The parameter of interest is the population of artists who are left-handed and its is 10% = (10/1000 = 0.10

b) The Null hypothesis and alternative hypothesis are;

H₀ : p = 0.10

H₁ : p > 0.10

c) The sample proportion is calculated as:

p = number of left handed artist / sample size

p = 26 / 200

p = 0.13

d) To find the p-value, The researcher should calculate the probability that the sample proportion would be 0.13 or larger for a sample of size 200 if the population proportion is actually 0.10.

reciprocal of dash and dash remains same​

Answers

Answer:

-1 and 1

Step-by-step explanation:

Reciprocal means "one divided by...".

1/-1 = -1 and 1/1 = 1

determine the results of the following operations​

Answers

Answer:

[tex]\sqrt[3]{4}\times (\sqrt[3]{16}-10 )[/tex]

Step-by-step explanation:

Let be [tex]\sqrt[3]{64}-\sqrt[3]{32} \times \sqrt[3]{125}[/tex], this expression is simplified as follows:

1) [tex]\sqrt[3]{64}-\sqrt[3]{32} \times \sqrt[3]{125}[/tex] Given

2) [tex]\sqrt[3]{4^{3}}-\sqrt[3]{2^{5}}\times \sqrt[3]{5^{3}}[/tex] Definition of power

3) [tex](4^{3})^{1/3}-(2^{2}\cdot 2^{3})^{1/3}\times (5^{3})^{1/3}[/tex] Definition of n-th root/[tex]a^{b+c}= a^{b}\cdot a^{c}[/tex]/[tex](a^{b})^{c} = a^{b\cdot c}[/tex]

4) [tex]4 - (2^{2})^{1/3}\times 2\times 5[/tex] [tex]a^{b+c}= a^{b}\cdot a^{c}[/tex]/[tex](a\cdot b)^{c} = a^{c}\cdot b^{c}[/tex]

5) [tex]4 - 10\times 4^{1/3}[/tex] Multiplication/Definition of power

6) [tex]4^{1/3}\cdot (4^{2/3}-10)[/tex] Distributive property/[tex]a^{b+c}= a^{b}\cdot a^{c}[/tex]

7) [tex]\sqrt[3]{4}\times [(4^{2})^{1/3}-10][/tex] [tex](a^{b})^{c} = a^{b\cdot c}[/tex]/Definition of n-th root

8) [tex]\sqrt[3]{4}\times (\sqrt[3]{16}-10 )[/tex] Definition of power/Result

What is the area of the house (including the drawing room, TV room, balcony, hallway, kitchen and bedroom)?

Answers

Answer:

The area of the house is A. 1,108

PLEASE HELP MEEEEEEEEEEEEEEE

Answers

Answer:

x=16.1

Step-by-step explanation:

open the brackets

-4.5= -0.5x-3.55

Take 3.55 to the other side.

-4.5-3.55 = -8.05

5/10x= -805/100

0.5x= - 8.05 = 16.1

One number is 4 times a first number. A third number is 100 more than the first number. If the sum of the three numbers is 370, find the numbers.
The three numbers are
(Use a comma to separate answers as needed.)

Answers

Answer:

  45, 180, 145

Step-by-step explanation:

Let n represent the first number. Then "one number" is 4n, and the third number is n+100. The sum of the three numbers is ...

  n + 4n + (n+100) = 370

  6n = 270

  n = 45

  4n = 180

  n+100 = 145

The three numbers are 45, 180, 145.

find x and y on triangle
Also the degree is 30 and the other thing is 7sqrt3​

Answers

Answer:

y =7

x =14

Step-by-step explanation:

Since this is a right triangle we can use trig functions

tan 30 = opp /adj

tan 30 = y/ 7 sqrt(3)

7 sqrt(3)  tan 30 = y

7 sqrt(3) * 1/ sqrt(3) =t

7 =y

sin 30 = opp/ hyp

sin 30 = 7/x

x sin 30 =7

x = 7/ sin 30

x = 7 / 1/2

x = 14

Question 6. Which expression is equivalent to 3x(x-6)+5(x-4)?

Answers

Answer:

8x- 38

Step-by-step explanation:

3(x-6)+5(x-4)

Distribute

3x- 18+5x -20

Combine like terms

3x+5x -18-20

8x- 38

Answer:

[tex]3(x-6)+5(x-4)[/tex]

[tex]3x-18+5x-20[/tex]

[tex]=(3+5)x-(20x+18)[/tex]

Add:- 3+5= 8, and 20+18= 38

[tex]=8x-38[/tex]

OAmalOHopeO

Parallelogram QRST has vertices Q(- 4, 2) . R(-2,4),5(0)) draw and label the image after a counterclockwise rotation of 270 degrees about the origia.please I need help.

Answers

Answer:

gr,wrgñegetjj

Step-by-step explanation:

jyyjytjjttj

What is the quotient of the synthetic division problem below, written in
polynomial form?
5)2 1 -55
O A. -2x+11
O B. -2x+9
O C. 2x+11
O D. 2x+9

Answers

Answer:

C. 2x+11

Step-by-step explanation:

2x+11≈21 -55

2x+11≈21 -55

2x+11≈21 -55

2x+11≈21 -55

2x+11≈21 -55

Other Questions
How does the teacher know you have completed your assignment in Google Classroom? the balance of payment between Vietnam and Iran The illustration below shows the graph of y as a function of x.Complete the following sentences based on the graph of the function.The y-intercept of the graph is the function value y=_____The smallest positive x-intercept of the graph is located at x=_____The greatest value of y is y=____ and it occurs when x=____For x between x and x= 2 the function value y___ 0 What are the odds IN FAVOR of picking a red marble from a bag of 10 green marbles, 10 yellow marbles, and 5 red marbles? Tickets for the front section to a rock concert cost $25 each. The back section tickets sold for $15 each. If 400 tickets were sold for a total revenue of $7,500, how many of each each type of ticket were sold? 1. Front 145, Back 255 2. Front 140, Back 260 3. Front 155, Back 245 4. Front 150, Back 250 Which of the following accurately describes this element? how does the mask help if you well? What structure(s) in the cardiovascular system connect arteries to veins?valvesheartcapillarieslymph A teacher took a group of 48 students to the movie theater. They have saved $353 in a fund for the students' tickets. How much money will they have left in the fund if the ticket for each student was $7 Of the 600 people at a music festival, 420 had attended the previous year. How many peopleout of 100 had attended the previous year? How do you get rid of acne and bacne..? I've tried daily washes for weeks to months now and my bacne just gets worse. My acne stays the exact same! How many times greater is6.6 x 10^10than3 x 10^72.22210002200 An atom of 120In has a mass of 119.907890 amu. Calculate the mass defect (deficit) in amu/atom. Use the masses: mass of 1H atom which civilization had the first democracy Jacobs age is two years more than the sum of the ages of his siblings Becky and Micah. Which equation represents Jacobs age? A. z = x + y 2; x represents Micah's age, y represents Becky's age, and z represents Jacob's age B. x = y + z + 2; x represents Jacob's age, y represents Micah's age, and z represents Becky's age C. x = 2 + y + z; x represents Becky's age, y represents Jacob's age, and z represents Micah's age D. y = x + z 2; x represents Jacob's age, y represents Becky's age, and z represents Micah's age Primary source: Zimmermann Telegram1.) What can you infer from the primary source about the purpose of this piece of literature?2.) How did the author intend for his work to be taken and used?3.) Was he trying to be controversial or calming?4.) How does it all influence the meaning of the piece? factorize 12p2q -9q2 This is the place.I saw her last times (into relative clause) how does liberty's daughters by mary beth norton end * Explain about monohybrid and diyhybrib cross with the help of punnet square